Difference between revisions of "1999 AMC 8 Problems/Problem 21"
Talkinaway (talk | contribs) (Solution 2) |
|||
Line 17: | Line 17: | ||
<math>\text{(A)}\ 20 \qquad \text{(B)}\ 30 \qquad \text{(C)}\ 35 \qquad \text{(D)}\ 40 \qquad \text{(E)}\ 45</math> | <math>\text{(A)}\ 20 \qquad \text{(B)}\ 30 \qquad \text{(C)}\ 35 \qquad \text{(D)}\ 40 \qquad \text{(E)}\ 45</math> | ||
− | ==Solution== | + | ==Solution 1== |
<asy> | <asy> | ||
unitsize(12); | unitsize(12); | ||
Line 31: | Line 31: | ||
</asy> | </asy> | ||
Note that <math>\angle B=180-100-40=40^\circ</math>. So <math>\angle A=180-110-40=\boxed{30^\circ}</math>. | Note that <math>\angle B=180-100-40=40^\circ</math>. So <math>\angle A=180-110-40=\boxed{30^\circ}</math>. | ||
+ | |||
+ | ==Solution 2== | ||
+ | |||
+ | Angle-chasing using the small triangles: | ||
+ | |||
+ | Use the line below and to the left of the <math>110^\circ</math> angle to find that the rightmost angle in the small lower-left triangle is <math>180 - 110 = 70^\circ</math>. | ||
+ | |||
+ | Then use the small lower-left triangle to find that the remaining angle in that triangle is <math>180 - 70 - 40 = 70^\circ</math>. | ||
+ | |||
+ | Use congruent vertical angles to find that the lower angle in the smallest triangle containing <math>A</math> is also <math>70^\circ</math>. | ||
+ | |||
+ | Next, use line segment <math>AB</math> to find that the other angle in the smallest triangle contianing <math>A</math> is <math>180 - 100 = 80^\circ</math>. | ||
+ | |||
+ | The small triangle containing <math>A</math> has a <math>70^\circ</math> angle and an <math>80^\circ</math> angle. The remaining angle must be <math>180 - 70 - 80 = \boxed{30^\circ, B}</math> |
Revision as of 16:14, 30 July 2011
Problem 21
The degree measure of angle is
Solution 1
Note that . So .
Solution 2
Angle-chasing using the small triangles:
Use the line below and to the left of the angle to find that the rightmost angle in the small lower-left triangle is .
Then use the small lower-left triangle to find that the remaining angle in that triangle is .
Use congruent vertical angles to find that the lower angle in the smallest triangle containing is also .
Next, use line segment to find that the other angle in the smallest triangle contianing is .
The small triangle containing has a angle and an angle. The remaining angle must be